Add solutions and graphics to Serway and Jewett v8's chapter 26 problems.
[course.git] / latex / problems / Serway_and_Jewett_8 / problem26.57.tex
1 \begin{problem*}{26.57}
2 A $2.00\U{nF}$ parallel-plate capacitor is charged to an initial
3 potential difference of $\Delta V_i=100\U{V}$ and is then isolated.
4 The dielectric material between the plates is mica, with a dielectric
5 constant of $5.00$.  \Part{a} How much work is required to withdraw
6 the mica sheet?  \Part{b} What is the potential difference across the
7 capacitor after the mica is withdrawn?
8 \end{problem*}
9
10 \begin{solution}
11 \Part{a}
12 The capacitance of a parallel plate capacitor is
13 \begin{equation}
14   C = \frac{\kappa\varepsilon_0 A}{d} \;.
15 \end{equation}
16 For other geometries, the constants change, but the capacitance is
17 always proportional to $\kappa$, the dielectric constant of the gap.
18 After removing the mica sheet, the capacitor will have a new
19 capacitance of
20 \begin{equation}
21   C' = \frac{\kappa'}{\kappa} C = \frac{1.00}{5.00} \cdot 2.00\U{nF}
22     = 400\U{pF} \;,
23 \end{equation}
24 where $\kappa' \approx 1.00$ is the dielectric constant for air (which
25 replaces the mica).
26
27 During the withdrawing process, the capacitance changes, and the
28 voltage between the plates changes, but because the capacitor is
29 isolated, the charge does not change.  We can use the initial voltage
30 to find that charge.
31 \begin{equation}
32   Q = C \Delta V_i = (2.00\E{-9}\U{F}) \cdot (100\U{V}) = 200\U{nC}
33 \end{equation}
34
35 Conserving energy during the withdrawing process,
36 \begin{align}
37   U_i + W &= U_f \\
38   \frac{Q^2}{2C} + W &= \frac{Q^2}{2C'} \\
39   W &= \frac{Q^2}{2} \cdot \p({\frac{1}{C'} - \frac{1}{C}})
40     = \frac{Q^2}{2} \cdot \p({\frac{\kappa}{\kappa' C} - \frac{1}{C}})
41     = \frac{Q^2}{2\kappa' C} \cdot (\kappa - \kappa')
42     = \frac{C\Delta V_i^2}{2\kappa'} \cdot (\kappa - \kappa') \\
43     &= \frac{(2.00\E{-9}\U{F}) \cdot (100\U{V})^2}{2 \cdot 1.00}
44        \cdot(5.00 - 1.00)
45     = \ans{40.0\U{$\mu$J}} \;.
46 \end{align}
47 \end{solution}